Tag Archives: DapAn

ĐỀ VÀ ĐÁP ÁN THI VÀO LỚP 10 CHUYÊN TOÁN SGD TPHCM NĂM 2022

Thời gian làm bài 150 phút

Bài 1: ( 1,0 điểm)
Cho $x, y$ là hai số thực thỏa mãn $x y+\sqrt{\left(1+x^2\right)\left(1+y^2\right)}=1$.
Tính giá trị của biểu thức $M=\left(x+\sqrt{1+y^2}\right)\left(y+\sqrt{1+x^2}\right)$.

Bài 2: (2,5 điểm)
a) Giải phương trình $\sqrt{x+4}+|x|=x^2-x-4$
b) Giải hệ phương trình $\left\{\begin{array}{l}\frac{x}{y+z}=2 x-1 \\\ \frac{y}{z+x}=3 y-1 \\\ \frac{z}{x+y}=5 z-1\end{array}\right.$

Bài 3: (1,5 điểm)
Cho hình vuông $A B C D$. Trên các cạnh $B C$ và $C D$ lần lượt lấy các điểm $M$ và $N$ sao cho $\widehat{M A N}=45^{\circ}$.
a) Chứng minh $M N$ tiếp xúc với đường tròn tâm $A$ bán kính $A B$.
b) Kė $M P$ song song với $A N$ ( $P$ thuộc đoạn $A B$ ) và kẻ $N Q$ song song với $A M$ ( $Q$ thuộc đoạn $A D$ ). Chứng minh $A P=A Q$.
Bài 4: (2,0 điếm)
Cho ba số thực dương $a, b, c$ thỏa $a+b+c=3$.
a) Chứng minh rằng $a b+b c+c a \leq 3$.
b) Tìm giá trị nhỏ nhất của biểu thức $P=\frac{a}{b^2+1}+\frac{b}{c^2+1}+\frac{c}{a^2+1}$.

Bài 5: (2,0 điểm)
Cho tam giác $A B C$ nhọn $(A B<A C)$ có các đường cao $A D, B E, C F$ cắt nhau tại $H$. Đường thẳng $E F$ cắt đường thẳng $B C$ tại $I$. Đường thẳng qua $A$ vuông góc với $I H$ tại $K$ và cắt $B C$ tại $M$.
a) Chứng minh tứ giác $I F K C$ nội tiếp và $\frac{B I}{B D}=\frac{C I}{C D}$.
b) Chứng minh $M$ là trung điểm của $B C$.

Bài 6: (1,0 điểm )
Số nguyên dương $n$ được gọi là “số tốt” nếu $n+1$ và $8 n+1$ đều là các số chính phương.
a) Hãy chỉ ra ví dụ ba “số tốt” lần lượt có $1,2,3$ chữ số.
b) Tìm các số nguyên $k$ thỏa mãn $|k| \leq 10$ và $4 n+k$ là hợp số với mọi $n$ là “số tốt”.

Đáp án do Star Education thực hiện

ĐỀ VÀ ĐÁP ÁN THI VÀO 10 CHUYÊN TOÁN SGD THÀNH PHỐ HỒ CHÍ MINH NĂM 2023

THỜI GIAN LÀM BÀI 150 PHÚT

Bài 1. (1,0 diểm) Cho $a, b$ là các số thực, $b \neq 0$ thỏa mãn điều kiện
$$
a^2+b^2=\frac{4 b^2}{\sqrt{a^2+b^2}+a}+a \sqrt{a^2+b^2}
$$

Tính giá trị của biểu thức $P=a^2+b^2$.
Bài 2. (2,5 điếm)
a) Giải phương trình: $x=\frac{5}{x-1}+2 \sqrt{x-2}$.
b) Giải hệ phương trình $\left\{\begin{array}{l}\frac{9 y+49}{x+y}+x+y=23 \\\ x \sqrt{x}+y \sqrt{y}=7(\sqrt{x}+\sqrt{y})\end{array}\right.$.

Bài 3. (2,5 điểm) Cho tam giác $A B C$ vuông tại $A(A B<A C)$, có đường cao $A H$. Dường tròn tâm $I$ nội tiếp tam giác $A B C$, tiếp xúc với các cạnh $B C, C A, A B$ lần lượt tại $D, E, F$. Gọi $J$ là giao điểm của $A I$ và $D E . K$ là trung điểm $A B$.
a) Chứng minh tứ giác $B I J D$ nội tiếp
b) Gọi $M$ là giao điểm của $K I$ và $A C, N$ là giao điểm của $A H$ và $E D$. Chứng minh $A M=A N$.
c) Gọi $Q$ là giao điểm của $D I$ và $E F, P$ là trung điểm của $B C$. Chứng minh ba điểm $A, P, Q$ thẳng hàng.

Bài 4. (2,0 diểm) Cho các số thực dương $x, y, z$ thỏa mãn $\sqrt{1+4 x y+2 x+2 y}+2 z=5$.
a) Chứng minh $\frac{1}{\sqrt{(2 x+1)(2 y+1)}}+\frac{1}{2 z+1} \geq \frac{2}{3}$.
b) Tìm giá trị nhỏ nhất của biễu thức $P=\frac{x+1}{2 x+1}+\frac{y+1}{2 y+1}+\frac{2 z+3}{4 z+2}$.

Bài 5. (1,0 điểm) Cho đường tròn tâm $O$ nội tiếp hình thoi $A B C D$. Gọi $E, F, G, H$ là các điểm lần lượt thuộc các cạnh $A B, B C, C D, D A$ sao cho $E F, G H$ cùng tiếp xúc với $(O)$.
a) Chứng minh $C G \cdot A H=A O^2$.
b) Chứng minh $E H$ song song $F G$.

Bài 6. (1,0 điểm) Xét các số nguyên $a<b<c$ thỏa mãn $n=a^3+b^3+c^3-3 a b c$ là số nguyên tố.
a) Chứng minh $a<0$.
b) Tìm tât cả các số nguyên $a, b, c(a<b<c)$ sao cho $n$ là một ước của 2023.

ĐÁP ÁN CỦA GIÁO VIÊN STAR EDUCATION

ĐỀ VÀ ĐÁP ÁN THI VÀO 10 CHUYÊN TOÁN TRƯỜNG PHỔ THÔNG NĂNG KHIẾU NĂM 2024

Thời gian làm bài: 150 phút

Đề bài:

Bài 1. (2 điểm)
1) Giải hệ phương trình $\left\{\begin{array}{l}x^3+z^3=y \\\ y^3+x^3=z \\\ z^3+y^3=x\end{array}\right.$.
2) Cho hai số nguyên dương $a, b$ phân biệt. Chứng minh phương trình sau có đúng ba nghiệm
$$
(\sqrt{x}-1)\left[x^2-2(a+b) x+a b+2\right]=0 .
$$

Bài 2. (1.5 điểm) Cho ba số thực $a, b, c$ không âm thóa mãn: $a^2+b^2+c^2+3=2(a b+b c+c a)$.
Chứng minh
$$
3 \leq a+b+c \leq \frac{2(a b+b c+c a)+3}{3} .
$$

Bài 3. (2 điểm) Với mỗi số tự nhiên $\mathrm{n}$, đặt $a_n=(2+\sqrt{3})^n+(2-\sqrt{3})^n$.
a) Chứng minh $a_{n+2}=4 a_{n+1}-a_n$ với mọi $n=0,1,2, \ldots$.
b) Tìm $\mathrm{n}$ để $a_n$ chia hết cho 4 .
c) Tìm $\mathrm{n}$ đề $a_n$ chia hết cho 14 .

Bài 4. (3 điểm) Cho tứ giác $A B C D$ nội tiếp đường tròn $(\mathrm{O})$ có tam giác $A B D$ là tam giác nhọn và đường chéo $\mathrm{AC}$ đi qua tâm $\mathrm{O}$ của đường tròn $(\mathrm{O})$. Gọi $\mathrm{I}$ là trung điểm $\mathrm{BD}, \mathrm{H}$ là trực tâm của tam giác $A B D$, $\mathrm{E}$ là giao điểm khác $\mathrm{A}$ của $\mathrm{AI}$ với $(\mathrm{O})$ và $\mathrm{K}$ là hình chiếu vuông góc của $\mathrm{H}$ lên $\mathrm{AI}$.
a) Chứng minh $C E H K$ là hình bình hành và $I B^2=I D^2=I A \cdot I K$.
b) Lấy điểm $\mathrm{F}$ trên cung nhỏ $\widehat{B D}$ của đường tròn $(\mathrm{O})$ sao cho $\widehat{B A F}=\widehat{D A I}$. Chứng minh các điểm $\mathrm{K}$ và $\mathrm{F}$ đối xứng nhau qua đường thẳng $\mathrm{BD}$.
c) Chứng minh các đường phân giác trong các góc $\widehat{B A D}$ và $\widehat{B K D}$ cắt nhau trên $\mathrm{BD}$.
d) Trên đường thẳng qua $\mathrm{H}$ và song song $\mathrm{AC}$ lấy điểm $\mathrm{T}$ sao cho $T H=T K$. Chứng minh các điểm $\mathrm{O}, \mathrm{K}, \mathrm{F}, \mathrm{T}$ cùng thuộc một đường tròn.

Bài 5. (1.5 điểm) Cho các sổ nguyên dương $a_1<a_2<a_3<\ldots<a_{30}<a_{31}$. Người ta ghi tất cả các số này lên 31 chiếc thẻ, mỗi thẻ ghi một số.
a) Biết rằng tổng các số được ghi trên 16 thẻ bất kỳ trong số 31 thẻ trên luôn lớn hơn tổng các số được ghi trên 15 thè còn lại. Chứng minh $a_1 \geq 226$.
b) Lấy $a_1, a_2, \ldots, a_{31}$ là 31 số nguyên dương đầu tiên: $1,2, \ldots, 31$. Người ta bỏ 31 thẻ được ghi các số này vào hai chiếc hộp một cách ngẫu nhiên. Khi kiểm tra một hộp thi thấy rằng trong hộp đó không có hai thẻ nào có tồng hai số được ghi là số chính phương. Chứng minh trong hộp còn lại ta có thể chọn ra được bốn thè và chia chúng thành hai cặp sao cho tổng hai sô̂ được ghi trên mỗi cặp là số chính phương.

Đáp án tham khảo từ Star Education

ĐỀ VÀ ĐÁP ÁN THI VÀO LỚP 10 CHUYÊN TOÁN TRƯỜNG PHỔ THÔNG NĂNG KHIẾU NĂM 2023

THỜI GIAN LÀM BÀI: 120 PHÚT

Bài 1. Giải hệ phương trình sau:
$$
\left\{\begin{array}{l}
(x+y)\left(4+\frac{1}{x y}\right)=1 \\\
\left(4 x+\frac{1}{x}\right)\left(4 y+\frac{1}{y}\right)=-20
\end{array}\right.
$$

Bài 2. Cho các số $a, b, c>0$ thỏa mãn $a b+b c+c a=a b c$.
a) Chứng minh rằng: $\frac{1}{\sqrt{a}}+\frac{1}{\sqrt{b}}+\frac{1}{\sqrt{c}} \leq \sqrt{3}$.
b) Chứng minh rằng: $(\sqrt{a}+\sqrt{b}+\sqrt{c})^2 \leq a b c \leq \frac{(a+b+c)^2}{3}$.

Bài 3. Cho bảng $4 \times 4$ được tô bằng ô đen hoặc trắng sao cho
i) mỗi hàng có số ô đen bằng nhau;
ii) mỗi cột có số ô đen đôi một khác nhau.
a) Tìm số ô đen ở mỗi hàng.
b) Một cặp ô được gọi là “tốt” khi có một ô đen và một ô trắng đứng cạnh nhau. Tìm số cặp tốt nhiều nhất tính theo hàng; số cặp tốt nhiều nhất tính theo cột.

Bài 4. Cho $m, n$ là các số nguyên không âm thỏa mãn $m^2-n=1$. Đặt $a=n^2-m$.
a) Chứng minh rằng $a$ là số lẻ.
b) Giả sử $a=3 \cdot 2^k+1, k$ là số nguyên không âm. Chứng minh rằng $k=1$.
c) Chứng minh rằng $a$ không là số chính phương.

Bài 5. Cho tam giác $A B C$ có đường tròn nội tiếp $(I) . D, E, F$ lần lượt là các tiếp điểm của $(I)$ với $B C, C A, A B$. Gọi $L$ là chân đường phân giác ngoài của $\angle B A C$ $(L \in B C)$. Vẽ tiếp tuyến $L H$ với đường tròn $(I)(H \neq D$ là tiếp điểm).
a) Chứng minh đường tròn ngoại tiếp tam giác $H A L$ đi qua tâm $I$.
b) Chứng minh $\angle B A D=\angle C A H$.
c) $A H$ kéo dài cắt $(I)$ tại $K(K \neq H)$. Gọi $G$ là trọng tâm của tam giác $K E F . D G$ cắt $E F$ tại $J$. Chứng minh rằng $K J \perp E F$.
d) Gọi $S$ là trung điểm $B C, K J$ cắt $(I)$ tại $R(R \neq K)$. Chứng minh rằng $A S, I R, E F$ dồng quy.

ĐÁP ÁN ĐƯỢC THỰC HIỆN BỞI STAR EDUCATION

ĐỀ và ĐÁP ÁN THI VÀO LỚP 10 CHUYÊN TOÁN TRƯỜNG PHỔ THÔNG NĂNG KHIẾU NĂM 2022

Bài 1. Cho hai phương trình: $x^2-2 a x+3 a=0 \quad$ (1) và $x^2-4 x+a=0$
a) Chứng minh ít nhất một trong hai phương trình trên có nghiệm.
b) Giả sử hai phương trình đều có hai nghiệm phân biệt. $T_1, T_2$ là tổng bình phương các nghiệm của (1) và $(2)$. Chứng minh $T_1+5 T_2>68$

Bài 2. Cho các số dương $a \geq b \geq c$ thỏa $a^2+b^2+c^2=1$. Chứng minh:
$$
\sqrt{4+(b+c)^2} \leq 2 a+b+c \leq \sqrt{4+4 a^2}
$$

Bài 3. Cho phương trình: $2^x+5^y=k^2\left(x ; y ; k \in \mathbb{N}^*\right)$
a) Chứng minh phương trình trên vô nghiệm khi $y$ là số chẵn.
b) Tìm $k$ để phương trình có nghiệm.

Bài 4. Cho tam giác $A B C$ có trực tâm $H, D$ đối xứng với $H$ qua $A$. $I$ là trung điểm của $C D$, đường tròn $(I)$ đường kính $C D$ cắt $A B$ tại $E, F(E$ thuộc tia $A B)$
a) Chứng minh $\angle E C D=\angle F C H$ và $A E=A F$.
b) Chứng minh $H$ là trực tâm của $\triangle C E F$.
c) $B H$ cắt $A C$ tại $K$. Chứng minh $E F K H$ nội tiếp và $E F$ là tiếp tuyến chung của $(C K E)$ và $(C K F)$.
d) Chứng minh tiếp tuyến tại $C$ của $(I)$ và tiếp tuyến tại $K$ của $(K E F)$ cắt nhau trên đường thẳng $A B$.

Bài 5. Cho dãy số nguyên $a_1 \geq a_2 \geq a_3 \geq \ldots \geq a_{21} \geq a_{22}$ thỏa mãn:
i) $\left|a_i\right| \leq 11$ và $a_i \neq 0 \forall i=1 ; 2 ; \ldots ; 22$
ii) $a_1+a_2+a_3+\ldots+a_{22}=1$
a) Chứng minh: $a_1 ; a_2>0$
b) Chứng minh có thể chọn $k \geq 1$ số từ $a_2 ; a_3 ; \ldots ; a_{22}$ để tổng $S$ của chúng thỏa $-10 \leq a_1+S \leq 0$.
c) Chứng minh từ dãy đã cho có thể chọn $n \geq 1$ số có tổng bằng 0 .

Đề thi và đáp án chọn đội dự tuyển PTNK năm 2022

Thời gian làm bài 120 phút.

Bài 1. Cho $a, b, c \geq 0$ thỏa $a^2+b^2+c^2=1$. Tìm giá trị lớn nhất và giả trị nhỏ nhất của biểu thức $P=a b+b c+c a-2(a+b+c)$.

Bài 2. Cho $k, n \in Z^{+}$, có bao nhiêu đơn ánh từ $\{1, 2, \cdots, 2k+1\} \to \{1, 2, \cdots, 2n\}$ thỏa $f(1) < f(2) < \ldots < f(k) < f(k+1) > f(k+2)>\ldots> f(2 k)>f(2 k+1)$ và $f(k+1) \neq 2 n-2$.

Bài 3. Cho $n$ là số nguyên dương, kí hiệu $a(n)=1+2+\ldots+n$ và $b(n)=1^2+2^2+\ldots+n^2$. Hỏi có tồn tại số $n$ sao cho $2(n+1) a(n)+3 b(n)-1$ là số chính phương?

Bài 4. Cho tam giác $A B C$ có $2 A=5 B=10 C$. Phân giác trong $B D$ cẳt trung tuyển $C M$ tại I. Một đường thẳng $d$ đi qua $D$ vuông góc với $A C$ cắt $B C$ và $A I$ lần lượt tại $E$ và $K . A E$ cắt $C K$ tại $F$. Chứng minh: $M F$ song song $B K$.

Lời giải tham khảo

Bài 1. Đặt $t=a+b+c$ ta có $a(1-a) \geq 0, b(1-b) \geq 0, c(1-c) \geq$, suy ra $a+b+c \geq$ $a^2+b^2+c^2=1$, và $(a+b+c)^2 \leq 3\left(a^2+b^2+c^2\right)=3$, suy ra $a+b+c \leq \sqrt{3}$ Ta có $1=(a+b+c)^2-2(a b+b c+a c) \Rightarrow a b+b c+c a=\frac{t^2-1}{2}$.
Do đó $P=\frac{t^2-1}{2}-2 t=\frac{1}{2} t^2-2 t-\frac{1}{2}$ với $1 \leq t \leq \sqrt{3}$.
Khảo sát hàm bậc hai trong đoạn ta có $\max P=-2$ khi $t=1$ và $\min P=1-2 \sqrt{3}$.
Vậy $\max P=-2$ khi $a=1, b=c=0$ và min $P=1-2 \sqrt{3}$ khi $a=b=c=\frac{1}{\sqrt{3}}$.

Bài 2. Do đó $f$ là đơn ánh, $\operatorname{Im} f$ là một tập con có $2 k+1$ phần tử của $A$, mặt khác $f(k+1)$ là giá trị lớn nhất nên $\operatorname{Im} f$ có giá trị lớn nhất khác $2 n-2$.
Ta đếm số tập con có $2 k+1$ phần tử của $A$ mà phần tử lớn nhất khác $2 n-2$. Số tập con có $2 k+1$ của $A$ là $C_{2 n}^{2 k+1}$, số tập con có $2 k+1$ mà có phần tử lớn nhất $2 n-2$ là bằng với số tập con có $2 k$ phần tử của ${1,2, \cdots 2 k-3}$, là $C_{2 n-3}^{2 k}$.
Do đó theo nguyên lí bù trừ số tập con có $2 k+1$ của tập $A$ mà phần tử lớn nhất khác $2 n-2$ là $\left(C_{2 n}^{2 k+1}-C_{2 n-3}^{2 k}\right)$.
Tiếp theo ta đếm số đơn ánh từ ${1,2, \cdots, 2 k+1}$ tới $A^{\prime}=\left\{a_1, a_2, \cdots, a_{2 k+1}\right\}$ thỏa đề bài, ta có $f(k+1)=a_{2 k+1}$, nên số đơn ánh bằng số cách chọn $k$ phần tử từ $A^{\prime}$ nên bằng $C_{2 k}^k$.
Vậy số đơn ánh thỏa đề bài $C_{2 k}^k\left(C_{2 n}^{2 k+1}-C_{2 n-3}^{2 k}\right)$

Bài 3. Ta có $a(n)=\frac{n(n+1)}{2}, b(n)=\frac{n(n+1)(2 n+1)}{6}$
Khi đó $P(n)=2(n+1) a(n)+3 b(n)-1=\frac{n(n+1)(4 n+3)}{2}-1$.
Giả sử $P(n)$ là số chính phương ta có $n(n+1)(4 n+3)=2\left(x^2+1\right)$, ta có $n(n+1)(4 n+3)$ luôn có ước nguyên tố dạng $p=4 k+3$, suy ra $p \mid 2\left(x^2+1\right)$ suy ra $p|x, p| 1$, vô lí! Vậy không tồn tại $n$ để $P(n)$ là số chính phương.

Bài 4.

Ta tính được $\angle A=\frac{5 \pi}{8}, \angle B=\frac{\pi}{4}, \angle C=\frac{\pi}{8}$. Vẽ đường cao $A N, N$ thuộc $B C$.
Ta có $\frac{B N}{N C}=\frac{A N}{N C}=\frac{\sin C}{\cos C}$ và $\frac{A D}{C D}=\frac{A B}{B C}=\frac{\sin C}{\sin 5 C}, \sin 5 C=\cos C$, suy ra $\frac{B N}{N C}=\frac{A D}{C D}$, do đó $A N, B D, C M$ đồng quy tại $I$ và $D N | A B$.
Ta có $\angle B A N=\angle A N D=\angle A C K=2 \angle A C K$, suy ra $A C K$ cân và $N$ là trung điểm $A K$, từ đó tam giác $A B K$ vuông cân.
Khi đó $\angle F N K=\angle A C K=45^{\circ}=\angle A K B$ và $\angle A N M=45^{\circ}$, do đó $M, N, F$ thẳng hàng và $M F | B K$.

Đề thi Học sinh giỏi Quốc gia năm 2021 (VMO 2021)

Ngày thi thứ nhất. Thời gian làm bài 180 phút.

Bài 1 (5 điểm). Cho dãy số thực $\left(x_n\right)$ có $x_1 \in\left(0, \frac{1}{2}\right)$ và $x_{n+1}=3 x_n^2-2 n x_n^3$ với mọi $n \geq 1$.
a) Chứng minh $\lim x_n=0$.
b) Với mỗi $n \geq 1$ đặt $y_n=x_1+2 x_2+\cdots+n x_n$. Chứng minh rằng dãy $\left(y_n\right)$ có giới hạn hữu hạn.

Bài 2 (5 điểm). Tìm tất cả các hàm số $f: \mathbb{R} \rightarrow \mathbb{R}$ thỏa mãn
$$
f(x) f(y)=f(x y-1)+x f(y)+y f(x)
$$
với mọi số thực $x, y$.
Bài 3 (5 điểm). Cho tam giác nhọn không cân $A B C$ có trực tâm $H$ và $D, E, F$ lần lượt là chân đường cao hạ từ các đỉnh $A, B, C$. Gọi $(I)$ là đường tròn ngoại tiếp tam giác $H E F$ với tâm $I$ và $K, J$ lần lượt là trung điểm $B C, E F$. Cho $H J$ cắt lại $(I)$ tại $G$, $G K$ cắt lại $(I)$ tại $L$.
a) Chứng minh rằng $A L$ vuông góc với $E F$.
b) Cho $A L$ cắt $E F$ tại $M, I M$ cắt lại đường tròn ngoại tiếp tam giác $I E F$ tại $N$, $D N$ cắt $A B, A C$ lần lượt tại $P, Q$. Chứng minh rằng $P E, Q F, A K$ dồng quy.

Bài 4(5 điểm). Với số nguyên $n \geq 2$, gọi $s(n)$ là tổng các số nguyên dương không vượt quá $n$ và không nguyên tố cùng nhau với $n$.
a) Chứng minh $s(n)=\frac{n}{2}(n+1-\varphi(n))$, trong đó $\varphi(n)$ là số các số nguyên dương không vượt quá $n$ và nguyên tố cùng nhau với $n$.
b) Chứng minh rằng không tồn tại số nguyên $n \geq 2$ thỏa mãn $s(n)=s(n+2021)$.

Ngày thi thứ 2. Thời gian làm bài 180 phút.

Bài 5 (6 điểm). Cho đa thức $P(x)=a_{21} x^{21}+a_{20} x^{20}+\cdots+a_1 x+a_0$ có các hệ số thuộc $[1011,2021]$. Biết rằng $P(x)$ có nghiệm nguyên và $c$ là một số dương sao cho $\left|a_{k+2}-a_k\right| \leq c$ với mọi $k \in{0,1, \ldots, 19}$.
a) Chứng minh rằng $P(x)$ có đúng một nghiệm nguyên.
b) Chứng minh $\sum_{k=0}^{10}\left(a_{2 k+1}-a_{2 k}\right)^2 \leq 440 c^2$.

Bài 6 (7 điểm). Một học sinh chia tất cả 30 viên bi vào 5 cái hộp được đánh số $1,2,3,4,5$ (sau khi chia có thể có hộp không có viên bi nào).
a) Hỏi có bao nhiêu cách chia các viên bi vào các hộp (hai cách chia là khác nhau nếu có một hộp có số bi trong hai cách chia là khác nhau)?
b) Sau khi chia, học sinh này sơn 30 viên bi đó bởi một số màu (mỗi viên được sơn đúng một màu, một màu có thể sơn cho nhiều viên bi), sao cho không có 2 viên bi nào trong cùng một hộp có màu giống nhau và từ hai hộp bất kì không thể chọn ra được 8 viên bi được sơn bởi 4 màu. Chứng minh rằng với mọi cách chia, học sinh đều phải dùng không ít hơn 10 màu để sơn bi.
c) Hãy chỉ ra một cách chia sao cho với đúng 10 màu, học sinh có thể sơn bi thỏa mãn các điều kiện ở câu b).

Bài 7 (7 điểm). Cho tam giác nhọn không cân $A B C$ nội tiếp đường tròn $(O)$. Gọi $D$ là giao điểm hai tiếp tuyến của $(O)$ tại $B$ và $C$. Đường tròn đi qua $A$ và tiếp xúc với $B C$ tại $B$ cắt trung tuyến đi qua $A$ của tam giác $A B C$ tại $G$. Cho $B G, C G$ lần lượt cắt $C D, B D$ tại $E, F$.
a) Đường thẳng đi qua trung điểm của $B E$ và $C F$ lần lượt cắt $B F, C E$ tại $M, N$. Chứng minh rằng các điểm $A, D, M, N$ cùng thuộc một đường tròn.
b) Cho $A D, A G$ lần lượt cắt lại đường tròn ngoại tiếp các tam giác $D B C, G B C$ tại $H, K$. Trung trực của $H K, H E, H F$ lần lượt cắt $B C, C A, A B$ tại $R, P, Q$. Chứng minh rằng các điểm $R, P, Q$ thẳng hàng.

Lời giải tham khảo

Đề thi và đáp án học sinh giỏi quốc gia năm 2022 (VMO 2022)

Ngày thi thứ nhất. Thời gian làm bài 180 phút.

Bài 1 (5,0 điểm)
Cho $a$ là một số thực không âm và dãy số $\left(u_n\right)$ được xác định bởi
$$
u_1=6, u_{n+1}=\dfrac{2 n+a}{n}+\sqrt{\dfrac{n+a}{n} u_n+4}, \quad \forall n \geq 1 .
$$
a) Với $a=0$, chứng minh rằng $\left(u_n\right)$ có giới hạn hữu hạn và tìm giới hạn đó.
b) Với mọi $a \geq 0$, chứng minh rằng $\left(u_n\right)$ có giới hạn hữu hạn.

Bài 2 (5,0 điểm)
Tìm tất cả các hàm số $f:(0 ;+\infty) \rightarrow(0 ;+\infty)$ thoả mãn
$$
f\left(\dfrac{f(x)}{x}+y\right)=1+f(y), \forall x, y \in(0 ;+\infty) .
$$

Bài 3(5,0$ điểm)
Cho tam giác nhọn $A B C$. Các điểm $E, F$ lần lượt thay đổi trên tia đối của các tia $B A, C A$ sao cho $B F=C E(E \neq B, F \neq C)$. Gọi $M, N$ tương ứng là trung điểm của $B E, C F$ và $D$ là giao điểm của $B F$ với $C E$.
a) Gọi $I, J$ lần lượt là tâm đường tròn ngoại tiếp các tam giác $D B E, D C F$. Chứng minh rằng $M N$ song song với $I J$.
b) Gọi $K$ là trung điểm của $M N$ và $H$ là trực tâm của tam giác $A E F$. Chứng minh rằng $H K$ luôn đi qua một điểm cố định.

Bài 4 (5,0 điểm)
Với mỗi cặp số nguyên dương $(n, m)$ thoả mãn $n<m$, gọi $s(n, m)$ là số các số nguyên dương thuộc đoạn $[n ; m]$ và nguyên tố cùng nhau với $m$. Tìm tất cả các số nguyên dương $m \geq 2$ thoả mãn đồng thời hai điều kiện sau:
i) $\dfrac{s(n, m)}{m-n} \geq \frac{s(1, m)}{m}$ với mọi $n=1,2, \ldots, m-1$;
ii) $2022^m+1$ chia hết cho $m^2$.

Ngày thi thứ hai. Thời gian làm bài 180 phút.

Bài 5(6,0 điểm)
Cho $P(x)$ và $Q(x)$ là hai đa thức khác hằng, có hệ số là các số nguyên không âm, trong đó các hệ số của $P(x)$ đều không vượt quá 2021 và $Q(x)$ có ít nhất một hệ số lớn hơn 2021. Giả sử $P(2022)=Q(2022)$ và $P(x), Q(x)$ có chung nghiệm hữu tỷ $\dfrac{p}{q} \neq 0(p, q \in \mathbb{Z} ; p$ và $q$ nguyên tố cùng nhau). Chứng minh rằng $|p|+n|q| \leq Q(n)-P(n)$ với mọi $n=1,2, \ldots, 2021$.

Bài 6 (7,0 điểm)
Gieo 4 con súc sắc cân đối, đồng chất. Ký hiệu $x_i\left(1 \leq x_i \leq 6\right)$ là số chấm trên mặt xuất hiện của con súc sắc thứ $i(i=1,2,3,4)$.
a) Tính số các bộ $\left(x_1, x_2, x_3, x_4\right)$ có thể có.
b) Tính xác suất để có một số trong $x_1, x_2, x_3, x_4$ bằng tổng của ba số còn lại.
c) Tính xác suất để có thể chia $x_1, x_2, x_3, x_4$ thành hai nhóm có tổng bằng nhau.

Bài 7 (7,0 điểm)
Cho tam giác $A B C$ có $B, C$ cố định trên đường tròn $(O)$ ( $B C$ không đi qua tâm $O$ ) và điểm $A$ thay đổi trên cung lớn $\overparen{B C}$ sao cho $A B \neq A C$. Đường tròn nội tiếp $(I)$ của tam giác $A B C$ tiếp xúc với $B C$ tại $D$. Gọi $I_a$ là tâm đường tròn bàng tiếp góc $\widehat{B A C}, L$ là giao điểm của $I_a D$ với $O I$ và $E$ là điểm trên $(I)$ sao cho $D E$ song song với $A I$.
a) Đường thẳng $L E$ cắt đường thẳng $A I$ tại $F$. Chứng minh rằng $A F=A I$.
b) Trên đường tròn $(J)$ ngoại tiếp tam giác $I_a B C$ lấy điểm $M$ sao cho $I_a M$ song song với $A D, M D$ cắt lại $(J)$ tại $N$. Chứng minh rằng trung điểm $T$ của $M N$ luôn thuộc một đường tròn cố định.

Đáp án chính thức

(Nguồn: Bộ giáo dục Việt Nam)

Đề thi và đáp án kì thi chọn đội tuyển thi Quốc gia trường Phổ thông Năng khiếu năm học 2018 – 2019

ĐỀ THI

Ngày thi thứ nhất

Bài 1. Cho số nguyên $a>1$. Tìm giá trị lớn nhất của số thực $d$ sao cho tồn tại một cấp số cộng có công sai $d$, số hạng đầu tiên là $a$ và có đúng hai trong các số $a^2, a^3, a^4, a^5$ là những số hạng của cấp số cộng đó.

Bài 2. Cho $n$ số thực $x_1, x_2, \ldots, x_n$. Với mỗi $i \in{1,2, \ldots, n}$, gọi $a_i$ là số các chỉ số $j$ mà $\left|x_i-x_j\right| \leq 1$ và $b_i$ là số các chỉ số $j$ mà $\left|x_i-x_j\right| \leq 2$ ( $i$ và $j$ có thể bằng).

(a) Chứng minh rằng tồn tại $i$ để $b_i \leq 3 a_i$.

(b) Gọi $A$ là số cặp $(i, j)$ có thứ tự mà $\left|x_i-x_j\right| \leq 1$ và $B$ là số cặp $(i, j)$ có thứ tự mà $\left|x_i-x_j\right| \leq 2$ ( $i$ và $j$ có thể bằng nhau). Chứng minh rằng $B \leq 3 A$.

Bài 3. Cho $p$ là số tự nhiên. Xét phương trình nghiệm nguyên $x^3+x+p=y^2$.

(a) Tìm số nguyên tố $p$ nhỏ nhất dạng $4 k+1$ sao cho phương trình có nghiệm.

(b) Chứng minh rằng nếu $p$ là số chính phương thì phương trình trên có nghiệm nguyên dương.

Bài 4. Cho tam giác $A B C$ nhọn nội tiếp đường tròn $(O)$ với $B, C$ cố định và $A$ di động trên $(O)$. $D$ là trung điểm $B C$. Trên $A B$ lấy các điểm $M, P$ và trên $A C$ lấy các điểm $N, Q$ sao cho $D A=D P=D Q$, dồng thời $D M \perp A C, D N \perp A B$.

(a) Chứng minh rằng các điểm $M, N, P, Q$ cùng thuộc một đường tròn $(\mathcal{C})$ và $(\mathcal{C})$ luôn đi qua một điểm cố định.

(b) Chứng minh rằng tâm của $(\mathcal{C})$ luôn thuộc một đường tròn cố định.

 

Ngày thi thứ hai

Bài 5. Cho số thực $a \neq 0$. Tìm giới hạn của dãy số $\left(u_n\right)$ thoả mãn:

$\quad\quad\quad\quad\quad\quad\quad\quad u_1=0, u_{n+1}\left(u_n+a\right)=a+1, \forall n \in \mathbb{N}^*$

Bài 6. Tìm tất cả các hàm số $f: \mathbb{R}^{+} \rightarrow \mathbb{R}^{+}$thoả mãn điều kiện:

$\quad\quad\quad\quad f\left(x f\left(y^2\right)-y f\left(x^2\right)\right)=(y-x) f(x y) \forall x, y \in \mathbb{R}^{+}, x>y$

Bài 7. Cho $n=2018.2019$. Gọi $A$ là tập hợp các bộ $\left(a_1, a_2, \ldots, a_n\right)$ có thứ tự thoả mãn điều kiện $a_i \in{0,1} \forall i \in{1,2, \ldots, n}$ và $\sum_{i=1}^n a_i=2018^2$. Có bao nhiêu bộ $\left(a_1, a_2, \ldots, a_n\right)$ từ $A$ để $\sum_{i=1}^k a_i \geq \frac{a}{2}$ và $\sum_{i=n-k+1}^n a_i \geq \frac{k}{2} \forall k \in{1,2, \ldots, n}$ ?

Bài 8. Đường tròn $(\mathcal{C})$ tâm $I$ nội tiếp tam giác $A B C$ và tiếp xúc với các cạnh $A B, A C$ tại $E, F$. $A M, A N$ là các đường phân giác trong, phân giác ngoài của góc $\angle B A C(M, N$ nằm trên $B C)$. Gọi $d_M, d_N$ lần lượt là các tiếp tuyến của $(\mathcal{C})$ qua $M, N$ và khác $B C$.

(a) Chứng minh rằng $d_M, d_N, E F$ đồng quy tại điểm $D$.

(b) Lấy trên $A B, A C$ các điểm $P, Q$ thoả mãn $D P|A C, D Q| A B$. Gọi $R, S$ là trung điểm của $D E, D F$. Chứng minh rằng $I$ thuộc đường thẳng qua các trực tâm của hai tam giác $D P S, D Q R$.

 

LỜI GIẢI

Ngày thi thứ nhất

Bài 1. Cho số nguyên $a>1$. Tìm giá trị lớn nhất của số thực $d$ sao cho tồn tại một cấp số cộng có công sai $d$, số hạng đầu tiên là $a$ và có đúng hai trong các số $a^2, a^3, a^4, a^5$ là những số hạng của cấp số cônng đó.

Lời giải: Trước hết, ta chứng minh rằng $d=a^3-a$ thoả mãn điều kiện. Thật vậy, xét cấp số cộng có số hạng đầu là $a$ và công sai là $d=a^3-a$ thì

$\quad\quad\quad\quad\quad\quad\quad\quad\quad\quad \left\{\begin{array}{l}a^3=a+\left(a^3-a\right) \\ a^5=a+\left(a^3-a\right)\left(a^2+1\right)\end{array} .\right.$

Do đó $a^3, a^5$ cùng thuộc cấp số cộng có công sai $d=a^3-a$.

Giả sử rằng tồn tại giá trị $d>a^3-a$ thoả mãn điều kiện bài toán. Khi đó:

$\quad\quad\quad\quad\quad\quad\quad\quad\quad\quad a+d>a+a^3-a=a^3$

Dẫn đến hai số hạng thuộc cấp số cộng phải là $a^4$ và $a^5$. Lại để ý rằng $a>1$ nên có $a<a^4<a^5$, kết hợp lại thì phải tồn tại hai số nguyên dương $k<l$ sao cho:

$\quad\quad\quad\quad\quad\quad\quad\quad\quad\quad \left\{\begin{array}{l}a^4=a+k d \\ a^5=a+l d\end{array}\right.$

Từ đó $a(a+k d)=a+l d$ hay $d(l-a k)=a^2-a>0$.

Chú ý rằng ta có $d>0$ nên $l-a k>0$, hơn nữa $l-a k \in \mathbb{Z}$ nên $l-a k \geq 1$. Điều này dẫn đến $a^2-a \geq d>a^3-a$, vô lý do $a>1$.

Vậy giá trị lớn nhất của $d$ là $\max d=a^3-a$.

Bài 2. Cho $n$ số thực $x_1, x_2, \ldots, x_n$. Với mỗi $i \in{1,2, \ldots, n}$, gọi $a_i$ là số các chỉ số $j$ mà $\left|x_i-x_j\right| \leq 1$ và $b_i$ là số các chỉ số $j$ mà $\left|x_i-x_j\right| \leq 2(i$ và $j$ có thể bằng nhau).

(a) Chứng minh rằng tồn tại $i$ dể $b_i \leq 3 a_i$.

(b) Gọi $A$ là số cặp $(i, j)$ có thứ tự mà $\left|x_i-x_j\right| \leq 1$ và $B$ là số cặp $(i, j)$ có thứ tự mà $\left|x_i-x_j\right| \leq 2$ ( $i$ và $j$ có thể bằng nhau). Chứng minh rằng $B \leq 3 A$.

Lời giải . (a) Không mất tính tổng quát, giả sử $x_1 \leq x_2 \leq \ldots \leq x_n$.

Xét $k=\max [a_1, a_2, \ldots, a_n]$ và $a_i=k$, khi đó tồn tại $k$ số trong dãy là:

$\quad\quad\quad\quad x_u \leq x_{u+1} \leq \ldots \leq x_i \leq \ldots \leq x_v \text { với }\left|x_u-x_i\right|,\left|x_v-x_i\right| \leq 1 .$

Ngoài ra vì tính lớn nhất của $k$ nên $\left|x_{u-1}-x_i\right|>1,\left|x_{v+1}-x_i\right|>1$.

Trong $\left[x_u, x_v\right]$, có đúng $k$ chỉ số $j$ để $\left|x_j-x_i\right| \leq 1<2$. Còn trước $x_u$, xét hai số $x_r, x_s$ sao cho $x_r \leq x_s$ và $\left|x_r-x_i\right| \leq 2,\left|x_s-x_i\right| \leq 2$ thì:

$\quad\quad\quad\quad \left|x_r-x_s\right|=x_s-x_r=\left(x_i-x_r\right)-\left(x_i-x_s\right)<2-1=1$

nên sẽ có không quá $k$ số $j$ để $\left|x_j-x_i\right| \leq 2$ vì nếu ngược lại, sẽ có nhiều hơn $k$ số liên tiếp trong dãy cách nhau không quá 1 đơn vị, mâu thuẫn với tính lớn nhất của $k$. Tương tự với các số sau $x_v$, vì thế nên $b_i \leq 3 k$ kéo theo $b_i \leq 3 a_i$.

(b) Ta sẽ chứng minh bằng quy nạp theo $n$.

Với $n=1$ rõ ràng $A=B=1$ nên khẳng định hiển nhiên đúng. Giả sử kết quả đúng với $n \geq 1$, ta sẽ chứng minh nó cũng đúng với $n+1$.

Xét dãy số thực $T=\left(x_1, x_2, \ldots, x_{n+1}\right)$ bất kỳ và giả sử $x_1 \leq x_2 \leq \ldots \leq x_{n+1}$. Ký hiệu $A_T, B_T$ là số cặp có thứ tự các chỉ số $(i, j)$ tương ứng với định nghĩa của đề bài. Giả sử $k \geq 1$ là số lượng lớn nhất các số của $T$ được chứa trong một đoạn độ dài bằng 2 nào đó.

Gọi $x_i$ là số cuối cùng của dãy mà trong đoạn $\left[x_i-1, x_i+1\right]$ có chứa đúng $k$ số (kể cả $x_i$ ). Gọi $T^{\prime}$ là dãy mới sau khi bỏ $x_i$ đi. Khi đó, số lượng các số thuộc $T^{\prime}$ có trong $\left[x_i-1, x_i+1\right]$ là $k-1$, ngoài ra $x_i$ đã bị bỏ đi thuộc về đúng $2 k-1$ cặp của $A_T$.

Do đó: $A_T=A_{T^{\prime}}+2 k-1$.

Ta viết lại như sau

$\quad\quad\quad\quad \left[x_i-2 ; x_i+2\right]=\left[x_i-2 ; x_i-1\right] \cup\left[x_i-1 ; x_i+1\right] \cup\left[x_i+1 ; x_i+2\right]$

Trừ đoạn ở giữa thì hai đoạn đầu và cuối chứa tối đa $k$ phần tử của $T$. Hơn nữa, do định nghĩa số $x_i$ nên trong đoạn $\left[x_i+1 ; x_i+2\right]$ có tối đa $k-1$ phần tử của $T$. Từ đó có tối đa:

$\quad\quad\quad\quad\quad\quad\quad\quad\quad\quad\quad\quad 2(k-1)+k=3 k-2$

phần tử của $T$ (không tính $x_i$ ) thuộc $\left[x_i-2 ; x_i+2\right]$. Dẫn đến:

$\quad\quad\quad\quad\quad\quad B_T \leq 2(3 k-2)+1+B_{T^{\prime}}=3(2 k-1)+B_{T^{\prime}}$

Áp dụng giả thiết quy nạp, ta có $B_{T^{\prime}}<3 A_{T^{\prime}}$ nên từ các điều trên thì:

$\quad\quad\quad\quad B_T \leq 3(2 k-1)+B_{T^{\prime}}<3(2 k-1)+3 A_{T^{\prime}}=3\left(A_{T^{\prime}}+2 k-1\right)=3 A_T .$

Theo nguyên lý quy nạp, bài toán cũng đúng với $n+1$.

Vậy bài toán được chứng minh hoàn toàn.

Nhận xét. Bài toán này thật ra liên quan đến phương pháp xác suất trong tổ hợp, có thể xem tại quyển “The Probabilistic Method” của GS. Noga Alon. Ta xét một lời giải khác như sau:

(a) Chọn $i$ sao cho số các chỉ số $j$ để $\left|x_i-x_j\right| \leq 1$ là lớn nhất. Khi đó, số lượng chỉ số $j$ sao cho $x_j \in\left(x_i+1, x_i+2\right]$ tối đa là $a_i$, vì nếu không thì tồn tại $j$ để $a_j>a_i$. Tương tự, số lượng chỉ số $j$ sao cho $x_j \in\left[x_i-2, x_i-1\right)$ tối đa là $a_i$.

Chú ý rằng với các chỉ số $j$ để $\left|x_i-x_j\right| \leq 2$ thì ta có điều sau:

$\quad\quad\quad\quad x_j \in\left[x_i-2, x_i-1\right) \cup\left(x_i-1, x_i+1\right) \cup\left(x_i+1, x_i+2\right]$

Số lượng các chỉ số đó chính là $b_i$, dẫn đến $b_i \leq a_i+a_i+a_i=3 a_i$. Hơn nữa, nếu đẳng thức xảy ra, ta phải có mỗi đoạn (hay nửa khoảng) ở phân hoạch trên chứa chính xác $a_i$ chỉ số $j$ của $x_j$.

(b) Bài toán hiển nhiên đúng với $n=1$. Giả sử rằng tồn tại $n>1$ để kết luận không đúng, ta chọn $n$ nhỏ nhất. Ta cũng chọn $i$ sao cho $a_i$ lớn nhất.

Gọi $A^{\prime}, B^{\prime}$ tương ứng là số cặp chỉ số $(k, l)$ mà $\left|x_k-x_l\right| \leq 1$ và $\left|x_k-x_l\right| \leq 2$, trong đó $1 \leq k, l \leq n$ và $k, l \neq i$. Vì $n$ là phản ví dụ nhỏ nhất nên $B^{\prime} \leq 3 A^{\prime}$.

Các cặp chỉ số $(k, l)$ mà $k=i$ hoặc $l=i$ và $\left|x_k-x_l\right| \leq 1$ đều phải có dạng $(k, i)$ hoặc $(i, k)$ trong đó $k \neq i$ và $(i, i)$. Có tổng cộng $2\left(a_i-1\right)+1$ cặp như thế nên $A=A^{\prime}+2\left(a_i-1\right)+1$.

Tương tự thì $B=B^{\prime}+2\left(b_i-1\right)+1$. Do đó nếu $b_i \leq 3 a_i-1$ thì:

$\quad\quad\quad\quad B=B^{\prime}+2 b_i-1 \leq 3 A^{\prime}+2\left(3 a_i-1\right)-1=3\left(A^{\prime}+2 a_i-1\right)=3 A$

Điều này trái với việc $n$ là phản ví dụ nhỏ nhất. Do đó $b_i \geq 3 a_i$. Theo ý (a) thì $b_i \leq 3 a_i$, từ đây phải có $b_i=3 a_i$. Hơn nữa, số lượng chỉ số $j$ để thỏa mãn $x_j \in\left[x_i-2, x_i-1\right)$ hoặc $x_j \in\left(x_i+1, x_i+2\right]$ dều phải bằng $a_i$.

Với mỗi $j, j^{\prime}$ sao cho $x_j, x_{j^{\prime}} \in\left[x_i-2, x_i-1\right)$, ta có $\left|x_j-x_{j^{\prime}}\right|<1$, dẫn đến $a_j \geq a_i$. Mặt khác $a_i$ là lớn nhất có thể nên $a_j=a_i$. Tương tự, với mỗi $j$ sao cho $x_j \in\left(x_i+1, x_i+2\right]$ thì $a_j=a_i$. Như vậy với mọi $j$ sao cho $1<\left|x_i-x_j\right| \leq 2$ thì $a_j=a_i$. Cũng với cách chọn chỉ số $j$ đó, lập luận tương tự như những ý trên, ta cũng phải có $b_j=3 a_j$.

Xây dựng đồ thị $\mathcal{G}$ với các đỉnh được đánh số là $1,2, \ldots, n$ sao cho cặp đỉnh $(k, l)$ kề nhau khi và chỉ khi $1<\left|x_k-x_l\right| \leq 2$. Những lập luận trên cho thấy mọi đỉnh $j$ mà tồn tại một đường đi từ $i$ đến $j$ đều phải thỏa mãn $a_j=a_i$ và $b_j=3 a_j$. Gọi $\mathcal{X}$ là tập hợp tất cả các đỉnh $j$ sao cho tồn tại một đường đi từ $i$ dến $j$ trong $\mathcal{G}$. Đặt $\mathcal{Y}={1,2, \ldots, n} \backslash \mathcal{X}(\mathcal{Y}$ có thể rỗng $)$.

Bây giờ, gọi $A_y, B_y$ tương ứng là số cặp chỉ số $(k, l)$ có tính thứ tự, có thể bằng nhau mà $\left|x_k-x_l\right| \leq 1$ và $\left|x_k-x_l\right| \leq 2$, trong đó $k, l \in \mathcal{Y}$. Chú ý rằng $A_{\mathcal{Y}}=B_{\mathcal{Y}}=0$ nếu $\mathcal{Y}=\emptyset$. Bởi $n$ là phản ví dụ nhỏ nhất, ta phải có $B_{\mathcal{Y}} \leq 3 A_{\mathcal{Y}}$. Ta gọi $a_{y, k}$ và $b_{y, k}$ tương ứng là số chỉ số $j \in \mathcal{Y}$ mà $\left|x_j-x_k\right| \leq 1$ và $\left|x_j-x_k\right| \leq 2$. Định nghĩa tương tự $a_{\mathcal{X}, k}$ và $b_{\mathcal{X}, k}$.

Với mọi $k \in \mathcal{Y}$, dễ thấy $k$ không kề bất cứ đỉnh nào trong $\mathcal{X}$, vì vậy ta có được $b_{\mathcal{X}, k}=0$ và $b_k=b_{\mathcal{Y}, k}+a_{\mathcal{X}, k}$. Từ đây dẫn đến đẳng thức sau:

$\quad\quad\quad\quad\quad\quad B=\sum_{k \in \mathcal{X}} b_k+\sum_{k \in \mathcal{Y}} b_k=3 \sum_{k \in \mathcal{X}} a_k+\sum_{k \in \mathcal{Y}}\left(b_{y, k}+a_{\mathcal{X}, k}\right)$

Ta đồng thời có $\sum_{k \in \mathcal{Y}} b_{y, k}=B_{\mathcal{Y}} \leq 3 A_{\mathcal{Y}}$. Hơn nữa, ta cũng có được:

$\quad\quad A=\sum_{k \in \mathcal{X}} a_k+\sum_{k \in \mathcal{Y}} a_k=\sum_{k \in \mathcal{X}} a_k+\sum_{k \in \mathcal{Y}}\left(a_{\mathcal{Y}, k}+a_{\mathcal{X}, k}\right)=\sum_{k \in \mathbb{X}} a_k+A_{\mathcal{Y}}+\sum_{k \in \mathcal{Y}} a_{\mathcal{X}, k}$

Do đó:

$\quad\quad\quad\quad B \leq 3 A_{\mathcal{Y}}+\sum_{k \in \mathcal{Y}} a_{\mathcal{X}, k}+3 \sum_{k \in \mathcal{X}} a_k \leq 3\left(A_{\mathcal{Y}}+\sum_{k \in \mathcal{Y}} a_{\mathcal{X}, k}+\sum_{k \in \mathcal{X}} a_k\right)=3 A$

Điều này dẫn đến giả sử phản chứng là sai.

Vì vậy, với mọi số nguyên dương $n$, ta phải có $B \leq 3 A$. Bài toán kết thúc.

Bài 3. Cho $p$ là số tự nhiên. Xét phương trình nghiệm nguyên

$\quad\quad\quad\quad\quad\quad\quad\quad\quad\quad\quad\quad x^3+x+p=y^2 .$

(a) Tìm số nguyên tố $p$ nhỏ nhất dạng $4 k+1$ sao cho phương trình có nghiệm.

(b) Chứng minh rằng nếu $p$ là số chính phương thì phương trình trên có nghiệm nguyên dương.

Lời giải. (a) Các số nguyên tố có dạng $4 k+1$ là $5,13,17, \ldots$

Trước hết, ta thấy với $p=13$ thì $x^3+x+13=y^2$ có nghiệm là $(x ; y)=(4 ; 9)$. Ta sẽ chứng minh rằng phương trình $x^3+x+5=y^2$ không có nghiệm nguyên. Xét modulo 4. Có các khả năng sau xảy ra:

  • Khi $x$ chia 4 dư $0,1,2,3$, vế trái chia 4 lần lượt dư $1,3,3,3$.
  • Khi $y$ chia 4 dư $0,1,2,3$, vế phải chia 4 lần lượt dư $0,1,0,1$.

Do đó $y$ phải lẻ và $4 \mid x$. Viết biểu thức đã cho thành:

$\quad\quad\quad\quad\quad\quad\quad\quad\quad\quad (x+3)\left(x^2-3 x+10\right)=y^2+5^2$

Do $x+3 \equiv 3(\bmod 4)$ nên $x+3$ có ước nguyên tố $q \equiv 3(\bmod 4)$. Ta biết rằng với $a, b \in \mathbb{Z}$ thì $a^2+b^2$ chia hết cho số nguyên tố $q \equiv 3(\bmod 4)$ khi và chỉ khi $q \mid a$ và $q \mid b$. Từ đó thì $q \mid 5$ hay $q=5$, mâu thuẫn.

Vậy $p=13$ là số nguyên tố nhỏ nhất cần tìm.

(b) Trước hết, ta giới thiệu kết quả sau (còn gọi là định lý 4 số):

Bổ Đề. Với các số nguyên dương $a, b, c, d$ thoả mãn $a b=c d$ thì tồn tại các số nguyên dương $x, y, z, t$ sao cho $a=x y, b=z t, c=x z, d=y t$.

Chứng minh bổ đề. Đặt $k=\operatorname{gcd}(a, c)$ và viết $a=k a_1, c=k c_1$ thì rõ ràng $\operatorname{gcd}\left(a_1, c_1\right)=1$. Thay vào đề bài, ta có

$\quad\quad\quad\quad\quad\quad\quad\quad\quad\quad k a_1 b=k c_1 d \text { hay } a_1 b=c_1 d .$

Từ đây chú ý $a_1 \mid c_1 d$, nên $a_1 \mid d$, đặt $d=a_1 \ell$. Thay vào thì có $b=\ell c_1$. Từ đó, ta chọn $x=k, y=a_1, z=c_1, t=\ell$ thì có ngay điều phải chứng minh.

Quay lại bài toán, do $p$ là số chính phương nên đặt $p=a^2, a \in \mathbb{Z}$. Ta viết lại phương trình thành dạng:

$\quad\quad\quad\quad\quad\quad\quad\quad x^3+x+a^2=y^2 \text { hay } x\left(x^2+1\right)=(y-a)(y+a) .$

Áp dụng kết quả trên vào bài toán, ta thấy tồn tại các số nguyên dương $m, n, p, q$ để $x=m n, x^2+1=p q, y+a=m p, y-a=n q$. Từ đó:

$\quad\quad\quad\quad\quad\quad\quad\quad\quad\quad (m n)^2+1=p q \text { và } m p-n q=2 a \text {. }$

Xét dãy số $\left(u_n\right)$ xác định bởi $u_0=0, u_1=1, u_{n+2}=\alpha u_{n+1}+u_n$, trong đó $\alpha$ là hằng số mà ta sẽ chọn sau. Rõ ràng với mọi $n$ thì

$\quad\quad\quad\quad\quad\quad\quad\quad u_n^2-u_{n+1} u_{n-1}=(-1)^{n-1}\left(u_1^2-u_2 u_0\right)=(-1)^{n-1} .$

Khi đó, với $n$ chẵn thì $u_n^2-u_{n+1} u_{n-1}=-1$. Chọn $m n=u_{2 k}$. Ta có:

$\quad\quad\quad\quad\quad\quad\quad u_2=\alpha, u_3=\alpha^2+1, u_4=\alpha\left(\alpha^2+2\right), u_5=\alpha^4+3 \alpha^2+1$

Chọn $p=u_3, q=u_5, m n=u_4$ thì rõ ràng $(m n)^2+1=p q$. Bây giờ ta chỉ cần có được

$\quad\quad\quad\quad\quad\quad m u_3-n u_5=2 a \text { hay } m\left(\alpha^2+1\right)-n\left(\alpha^4+3 \alpha^2+1\right)=2 a .$

Từ đây chọn $\alpha=4 a^2$ và viết $m=2 a\left(\alpha^2+2\right), n=2 a\left(\alpha^4+3 \alpha^2+1\right)$ thì đẳng thức trên sẽ thoả mãn, vì

$\quad\quad\quad\quad\quad\quad\quad\quad \left(\alpha^2+1\right)\left(\alpha^2+2\right)-\left(\alpha^4+3 \alpha^2+1\right)=1 .$

Vậy phương trình có một cặp nghiệm cụ thể là

$\quad\quad (x, y)=\left(4 a^2\left(16 a^4+2\right), 2 a\left(16 a^4+2\right)\left(16 a^4+1\right)-a\right) \text { với } a=\sqrt{p} \in \mathbb{Z}^{+} .$

Bài 4. Cho tam giác $A B C$ nhọn nội tiếp đường tròn $(O)$ với $B, C$ cố định và $A$ di động trên $(O)$. $D$ là trung điểm $B C$. Trên $A B$ lấy các điểm $M, P$ và trên $A C$ lấy các điểm $N, Q$ sao cho $D A=D P=D Q$, dồng thời $D M \perp A C, D N \perp A B$.

(a) Chứng minh rằng các điểm $M, N, P, Q$ cùng thuộc một đường tròn $(\mathcal{C})$ và (C) luôn đi qua một điểm cố định.

(b) Chứng minh rằng tâm của $(\mathcal{C})$ luôn thuộc một đường tròn cố định.

Lời giải . (a) Dễ thấy tam giác $A M Q$ cân tại $M$ nên

$\quad\quad \angle D M Q=\angle D M A=90^{\circ}-\angle A=\frac{180^{\circ}-2 \angle A}{2}=\frac{180^{\circ}-\angle P D Q}{2}=\angle D P Q$

Do đó tứ giác $M P D Q$ nội tiếp. Chứng minh tương tự, ta có tứ giác $Q N D P$ nội tiếp nên $M, N, P, Q$ cùng thuộc một đường tròn $(\mathcal{C})$, và $(\mathcal{C})$ luôn đi qua điểm $D$ cố định.

(b) Gọi $K B, K C$ là hai tiếp tuyến của $(O)$. Ta có $D, K, O$ thẳng hàng, lại có:

$\quad\quad\quad\quad\quad \angle B K O=90^{\circ}-\angle B O K=90^{\circ}-\angle B A C=\angle B M D$

Từ đó tứ giác $B D K M$ nội tiếp. Để ý rằng $K D \perp B C$ nên $K M \perp A B$, hơn nữa $D N \perp A B$ nên $K M | D N$. Tương tự thì $K N | D M$. Do đó $D M K N$ là hình bình hành hay $D K, M N$ có $J$ là trung điểm chung.

Gọi $I$ là tâm của $(\mathcal{C})$ thì $I J \perp M N$ và $J L | A D$. Chú ý rằng $D$ là tâm $(A P Q)$ và cũng là trực tâm tam giác $A M N$ nên $P Q, M N$ là hai đường đối song. Đồng thời nếu $L$ là trung điểm $A D$ thì $J L$ vuông góc với đường nối hai chân đường cao từ $M, N$ của tam giác $A M N$ nên $J L \perp P Q$. Lại có $D P=D Q$ và $I P=I Q$ nên $I D \perp P Q$, do đó $J L | D I$.

Từ đây $I D L J$ là hình bình hành và $I L, D J$ có $T$ là trung điểm chung cố định. Xét phép vị tự tâm $D$ tỉ số $\frac{1}{2}$ hợp với phép đối xứng tâm $T$ thì $A \mapsto I$. Do $A$ thuộc đường tròn $(O)$ cố định nên $I$ cũng thuộc đường tròn cố định là ảnh của $(O)$ qua hợp các phép biến hình trên. Bài toán kết thúc.

Nhận xét. Bài toán này còn một hướng tiếp cận bản chất hơn như sau. Nếu gọi $A^{\prime}$ là điểm đối xứng của $A$ qua $D$ thì $K, A^{\prime}$ là hai điểm liên hợp đẳng giác trong tam giác $A B C$, từ đó đường tròn $(\mathcal{C})$ chính là đường tròn đi qua các hình chiếu của $K, A^{\prime}$ trên các cạnh tam giác $A B C$, dồng thời $I$ là trung diểm $K A^{\prime}$.

Dưới đây là một bài toán tương tự: Cho tam giác nhọn $A B C$ nội tiếp đường tròn $(O)$ có $B C$ cố định và $A$ di dộng trên $(O)$. Gọi $H$ là trực tâm tam giác và lấy điểm $E, F$ thuộc $A B, A C$ theo thứ tự đó sao cho $H$ là trung điểm $E F$.

  1. Chứng minh rằng tâm của đường tròn $(A E F)$ luôn thuộc một đường tròn cố định. Đặt là $\omega$.
  2. Giả sử $\omega$ cắt lại $(O)$ tại các điểm $X, Y$. Chứng minh rằng $X, Y, O$ thẳng hàng.

 

Ngày thi thứ hai

Bài 5. Cho số thực $a \neq 0$. Dãy số $\left(u_n\right)$ thoả mãn:

$\quad\quad\quad\quad\quad\quad\quad\quad\quad\quad u_1=0, u_{n+1}\left(u_n+a\right)=a+1 \forall n \in \mathbb{N}^*$

Tìm giới hạn của dãy số $\left(u_n\right)$.

Lời giải: Đặt $x_{n+1}=(a+1) y_n$ và $y_{n+1}=x_n+a y_n$. Ta có:

$\quad\quad\quad\quad\quad\quad\quad y_{n+2}=x_{n+1}+a y_{n+1}=a y_{n+1}+(a+1) y_n$

Đồng thời $u_n=\frac{x_n}{y_n}$. Để ý rằng $u_1=0, u_2=\frac{a+1}{a}$. Chọn $y_1=1, y_2=a$. Từ đó:

$\quad\quad\quad\quad\quad\quad\quad\quad\quad\quad y_n=\frac{(a+1)^n-(-1)^n}{a+2} \forall n \geq 1$

Công thức trên chỉ xác định với $a \neq-2$ nên xét trường hợp $a=-2$, ta có dãy

$\quad\quad\quad\quad\quad\quad\quad\quad\quad\quad \left\{\begin{array}{l}u_1=0, \\ u_{n+1}=\frac{1}{2-u_n}, n \geq 1\end{array} .\right.$

Bằng quy nạp, ta chứng minh được $u_n \in[0 ; 1)$ nên:

$\quad\quad\quad\quad\quad\quad\quad u_{n+1}-u_n=\frac{1}{2-u_n}-u_n=\frac{\left(u_n-1\right)^2}{2-u_n}>0$

Dãy $\left(u_n\right)$ tăng và bị chặn trên bởi 1 nên có giới hạn hữu hạn là $L \in(0,1)$. Giải phương trình giới hạn, ta có được $L=\frac{1}{2-L}$. Khi đó thì $L=1$.

Tiếp theo, xét $a \neq-2$, ta có:

$\quad\quad\quad\quad u_n=\frac{x_n}{y_n}=\frac{(a+1) y_{n-1}}{y_n}=\frac{(a+1)^n+(a+1)(-1)^n}{(a+1)^n-(-1)^n} \forall n \in \mathbb{N}^*$

Đặt $-(a+1)=b \in{-1 ; 1}$, ta viết lại thành:

$\quad\quad\quad\quad\quad\quad\quad\quad\quad\quad u_n=\frac{b^n-b}{b^n-1} \forall n \geq 1$

Có các khả năng sau xảy ra:

  • Nếu $b>1$ hoặc $b<-1$, tương ứng là $a<-2$ hoặc $a>0$, thì $\lim u_n=1$.
  • Nếu $-1<b<1$, tương ứng là $-2<a<0$, thì $\lim u_n=b=-(a+1)$.

Vậy ta có kết luận sau trong các trường hợp của $a$ :

  • Nếu $a \in(-2 ; 0)$ thì $\lim u_n=-(a+1)$.
  • Nếu $a \notin(-2 ; 0)$ thì $\lim u_n=-1$.

Bài 6. Tìm tất cả các hàm số $f: \mathbb{R}^{+} \rightarrow \mathbb{R}^{+}$thoả mãn diều kiện:

$\quad\quad\quad\quad\quad\quad f\left(x f\left(y^2\right)-y f\left(x^2\right)\right)=(y-x) f(x y) \forall x, y \in \mathbb{R}^{+}, x<y .$

Lời giải . Theo giả thiết thì với mọi $y>x>0$, ta đều có

$\quad\quad\quad\quad\quad\quad\quad\quad x f\left(y^2\right)-y f\left(x^2\right)>0 \Rightarrow \frac{f\left(y^2\right)}{f\left(x^2\right)}>\frac{y}{x}>1 .$

Do đó,

$\quad\quad\quad\quad\quad\quad\quad\quad y^2>x^2 \Leftrightarrow y>x \Leftrightarrow f\left(y^2\right)>f\left(x^2\right)$

nên hàm $f$ dã cho đồng biến trên $\mathbb{R}^{+}$. Trong đề bài, thay $y=x+1$, ta có

$\quad\quad\quad\quad\quad\quad\quad f\left(x f\left((x+1)^2\right)-(x+1) f\left(x^2\right)\right)=f(x(x+1))$

hay

$\quad\quad\quad\quad\quad\quad\quad\quad x f\left((x+1)^2\right)-(x+1) f\left(x^2\right)=x(x+1) $

$\quad\quad\quad\quad\quad\quad\quad\quad \Leftrightarrow \frac{f\left((x+1)^2\right)}{x+1}=\frac{f\left(x^2\right)}{x}+1, \forall x>0$

Thực hiện thao tác này nhiều lần, ta có

$\quad\quad\quad\quad\quad\quad\quad\quad \frac{f\left((x+n)^2\right)}{x+n}=\frac{f\left(x^2\right)}{x}+n, \forall x>0, n \in \mathbb{Z}^{+}$

hay

$\quad\quad\quad\quad\quad\quad\quad\quad x f\left((x+n)^2\right)-(x+n) f\left(x^2\right)=n x(x+n) .$

Trong dề bài, thay $y=x+n$, ta có

$\quad\quad\quad\quad\quad\quad f\left(x f\left((x+n)^2\right)-(x+n) f\left(x^2\right)\right)=n f(x(x+n)) $

$\quad\quad\quad\quad\quad\quad \Leftrightarrow f(n x(x+n))=n f(x(x+n)) .$

Với mọi $n \in \mathbb{Z}^{+}, y>0$, ta luôn chọn được $x>0$ để $x(x+n)=y$ nên ta có

$\quad\quad\quad\quad\quad\quad\quad\quad\quad\quad f(n y)=n f(y), \forall n \in \mathbb{Z}^{+}, y \in \mathbb{R}^{+} .$

Đặt $f(1)=a>0$, với mọi $n \in \mathbb{Z}^{+}$, cho $y=\frac{1}{n}$, suy ra

$\quad\quad\quad\quad\quad\quad\quad\quad\quad\quad f(1)=n f\left(\frac{1}{n}\right) \Rightarrow f\left(\frac{1}{n}\right)=\frac{a}{n} .$

Do đó,

$\quad\quad\quad\quad\quad\quad\quad\quad f\left(\frac{n}{m}\right)=n f\left(\frac{1}{m}\right)=\frac{n}{m} a, \forall m, n \in \mathbb{Z}^{+}$

hay $f(x)=a x, \forall x \in \mathbb{Q}^{+}$. Với mọi số thực $x_0>0$, chọn hai dãy số hữu tỷ $\left(a_n\right),\left(b_n\right)$ sao cho $a_n<x_0<b_n$ và $\lim a_n=\lim b_n=x_0$. Rõ ràng

$\quad\quad\quad\quad\quad\quad f\left(a_n\right)<f\left(x_0\right)<f\left(b_n\right) \Rightarrow a \cdot a_n<f\left(x_0\right)<a \cdot b_n,$

nên cho $n \rightarrow+\infty$, ta có $f\left(x_0\right)=a x_0$. Do đó, với mọi số thực $x>0$ thì $f(x)=a x$. Thay vào biểu thức đã cho, ta có

$\quad\quad\quad\quad\quad \left\{\begin{array}{l}f\left(x f\left(y^2\right)-y f\left(x^2\right)\right)=a^2\left(x y^2-x^2 y\right)=a^2(y-x) x y \\ (y-x) f(x y)=a(y-x) x y\end{array}\right.$

nên $a=1$. Vậy tất cả các hàm số cần tìm là $f(x)=x, \forall x>0$.

Nhận xét. Có một điều đáng chú ý ở bài toán này là việc từ giả thiết, ta phải ngầm hiểu rằng $x f\left(y^2\right)-y f\left(x^2\right)>0$ với mọi cặp số dương $x<y$. Ta có thể thêm tường minh điều kiện đó vào đề bài cho rõ. Tuy nhiên, nếu thêm theo kiểu như sau thì sẽ có một chút vấn đề phát sinh:

Tìm tất cả các hàm số $f: \mathbb{R}^{+} \rightarrow \mathbb{R}^{+}$thoả mãn với mọi cặp số dương $x<y$, nếu $x f\left(y^2\right)-y f\left(x^2\right)>0$ thì

$\quad\quad\quad\quad\quad\quad\quad\quad\quad\quad f()=(y-x) f(x y) \forall x, y \in \mathbb{R}^{+}, x<y .$

Khi đó, ta có thể nhận thêm một hàm số thỏa mãn nữa là $f(x)=\sqrt{x}$. Lý do là vì với mọi cặp số $y>x>0$, ta đều có $x f\left(y^2\right)-y f\left(x^2\right)=0$, mà vì thế, điều kiện “nếu” ở trên là sai nên mệnh đề kéo theo là đúng.

Bài 7. Cho $n=2018.2019$. Gọi $A$ là tập hợp các bộ $\left(a_1, a_2, \ldots, a_n\right)$ có thứ tự thoả mãn điều kiện $a_i \in{0,1} \forall i \in{1,2, \ldots, n}$ và $\sum_{i=1}^n a_i=2018^2$.

Có bao nhiêu bộ $\left(a_1, a_2, \ldots, a_n\right)$ từ $A$ dể:

$\quad\quad\quad\quad\quad \sum_{i=1}^k a_i \geq \frac{a}{2} \text { và } \sum_{i=n-k+1}^n a_i \geq \frac{k}{2} \forall k \in{1,2, \ldots, n}$

Lời giải. Ta giải bài toán tổng quát khi thay 2018 bởi $m \in \mathbb{Z}^{+}$. Bài toán đã cho tương đương với bài toán sau:

Trong hệ trục tọa độ Oxy, xét lưới điểm nguyên trong hình chũ nhật có đỉnh dưới bên trái là $O(0 ; 0)$ và dỉnh trên bên phải là $A\left(m^2 ; m\right)$. Dặt $B(m ; m)$ và $C\left(m^2-m ; 0\right)$, hỏi có bao nhiêu đương đi tù̀ $O \rightarrow A$ sao cho mỗi bước, ta đi sang phải hoặc lên trên 1 đơn vị, gọi là đương đi đơn, và không vượt lên trên $O B$ cũng nhu không xuống dưới $A C$ ?

Ở đây, các số $0 ; 1$ tương ứng với các bước đi lên trên, các bước đi sang phải; còn điều kiện tổng $k$ số đầu và tổng $k$ số cuối không nhỏ hơn $\frac{k}{2}$ tương ứng với số lượng bước đi lên không vượt quá số lượng bước đi sang phải. Để thuận tiện, ta gọi đường đi cắt $d$ nếu nó có các phần nằm về cả hai phía của $d$. Trước hết, ta sẽ chứng minh bổ đề sau:

Bổ Đề. Số đường đi đơn từ $O \rightarrow A(m ; n)$, có cắt đường thẳng $y=x$, là $C_{m+n}^{m+1}$.

Thật vậy, Xét đường thẳng $(d): y=x+1$, rõ ràng các đường đi đơn cắt $y=x$ dều sẽ có điểm chung với đường thẳng $(d)$ này. Tại các điểm chung đó, ta thực hiện đối xứng trục để được một đường đi mới xuất phát từ $O \rightarrow A^{\prime}(n-1, m+1)$.

Trong hình trên, đường cũ là đứt nét, còn đường mới là liền nét. Rõ ràng phép đối xứng trục trên là song ánh, biến các đường cần tìm (cắt $y=x$ ), thành các đường từ $O \rightarrow A^{\prime}$; do đó, số lượng đường cần tìm là $C_{m+n}^{n-1}$.

Trở lại bài toán,

Số đường đi đơn từ $O \rightarrow A\left(m^2 ; m\right)$ là $C_{m^2+m}^m$ vì nó bằng số cách chọn $m$ lần đi lên trong tổng số $m^2+m$ lần di chuyển, trong đó số đường đi cắt $O B$ bằng số đường đi cắt $A C$ và bằng $C_{m^2+m}^{m-1}$ (theo bổ đề).

Do đó, ta chỉ cần tìm số đường đi cắt cả $O B, A C$ với ý tưởng đối xứng hai lần đã dùng để chứng minh bổ đề.

Đầu tiên, ta thực hiện đối xứng qua đường thẳng $y=x+1$; khi đó, các đường đi đơn sẽ xuất phát từ $O \rightarrow A^{\prime}\left(m-1 ; m^2+1\right)$. Do các đường ban đầu còn vượt qua $A C$ nên các đường mới phải cắt thêm $y=x+m^2-m+3$. Tiếp tục đối xứng qua đường thẳng này, ta đưa về đếm số đường đi đơn từ $O \rightarrow A^{\prime \prime}\left(m-2, m^2+2\right)$. Suy ra số đường đi trong trường hợp này là $C_{m^2+m}^{m-2}$. Vậy theo nguyên lý bù trừ, kết quả cần tìm sẽ là

$\quad\quad\quad\quad\quad\quad\quad\quad\quad\quad C_{m^2+m}^m-2 C_{m^2+m}^{m-1}+C_{m^2+m}^{m-2} .$

Thay $m=2018$, ta có số lượng đường đi, cũng chính là số bộ thỏa mãn đề bài.

Nhận xét. Dưới đây là một số kết quả tương tự về đường đi đơn trong đề bài

$1$. Số đường đi đơn từ $(0 ; 0) \rightarrow(m ; n)$ mà không có điểm chung với $y=x$ là

$\quad\quad\quad\quad\quad\quad\quad\quad\quad\quad\quad\quad \frac{m-n}{m+n} C_{m+n}^m .$

$2$. Số đường đi đơn từ $(0 ; 0) \rightarrow(m ; n)$ mà không vượt qua $y=x$ là

$\quad\quad\quad\quad\quad\quad\quad\quad\quad\quad\quad C_{m+n}^n-C_{m+n}^{n-1} .$

$3$. Số đường đi gồm $n$ bước mà không vượt $y=x$ là

$\quad\quad\quad\quad\quad\quad\quad\quad\quad \sum_{i=n / 2}^n \frac{n !(2 i+1-n)}{(i+1) !(n-i) !}=C_n^{[n / 2]} .$

$4$. Số đường đi đơn từ $(0 ; 0) \rightarrow(m ; n)$ mà không có điểm chung với $y=x+t$ là

$\quad\quad\quad\quad\quad\quad\quad\quad\quad\quad\quad\quad C_{m+n}^n-C_{m+n}^{m-t}$.

Bạn đọc có thể dùng phương pháp tương tự trên để giải quyết các bài toán này.

Bài 8. Đường tròn $(\mathcal{C})$ tâm $I$ nội tiếp tam giác $A B C$ và tiếp xúc với các cạnh $A B, A C$ tại $E, F$. $A M, A N$ là các đường phân giác trong, phân giác ngoài của góc $\angle B A C(M, N$ nằm trên $B C)$. Gọi $d_M, d_N$ lần lượt là các tiếp tuyến của $(\mathcal{C})$ qua $M, N$ và khác $B C$.

(a) Chứng minh rằng $d_M, d_N, E F$ dồng quy tại điểm $D$.

(b) Lấy trên $A B, A C$ các điểm $P, Q$ thoả mãn $D P|A C, D Q| A B$. Gọi $R, S$ là trung điểm của $D E, D F$. Chứng minh rằng $I$ thuộc đường thẳng qua các trực tâm của hai tam giác $D P S, D Q R$.

Lời giải. (a) Gọi $X, Y$ lần lượt là tiếp điểm của tiếp tuyến thứ hai kẻ từ $M$ dến $(I)$ và $D^{\prime}$ là tiếp điểm của $(I)$ trên $B C$. Gọi $K$ là trung điểm $E F$.

Xét trong đường tròn $(I)$ thì $E F$ là đường đối cực của $A$ và $K \in E F$ nên đối cực của $K$ sẽ đi qua $A$, mà $N A \perp I A$ nên $N A$ chính là đường đối cực của $K$.

Đường đối cực của $K$ đi qua $N$ nên đối cực của $N$, là $D^{\prime} Y$, sẽ đi qua $K$. Dễ thấy rằng $A M$ là trục đối xứng của tứ giác $D^{\prime} X E F$ nên suy ra $D^{\prime} X | E F$. Xét $D^{\prime}(E F, X Y)$, ta có có $D^{\prime} Y$ đi qua trung điểm của $E F$ và $D^{\prime} X | E F$ nên

$\quad\quad\quad\quad\quad\quad\quad\quad\quad\quad D^{\prime}(E F, X Y)=-1$

hay tứ giác $E X F Y$ điều hòa. Suy ra $M X, N Y, E F$ đồng quy. Ngoài ra ta cũng có $X, Y, A$ thẳng hàng.

(b) Dễ thấy các tam giác $P E D$ và $D Q F$ là các tam giác cân. Gọi $H_1, H_2$ lần lượt là trực tâm của tam giác $\triangle D P S, \triangle D Q R$. Ta có

$\quad\quad\quad\quad\quad\quad\quad\quad \angle P H_1 S=\angle P D F=\angle A F E=\angle P E S$

nên $E P S H_1$ là tứ giác nội tiếp. Suy ra $R H_1 \cdot R P=R S \cdot R E$. Ngoài ra,

$\quad\quad\quad\quad\quad\quad\quad\quad\quad\quad K A \cdot K I=K E \cdot K F$

nên

$\quad\quad\quad\quad\quad\quad\quad\quad\quad \frac{R P}{K A} \cdot \frac{R H_1}{K I}=\frac{R E}{K E} \cdot \frac{R S}{K F} .$

Theo định lý Thales thì $\frac{R P}{K A}=\frac{R E}{K E}$ nên $\frac{R H_1}{K I}=\frac{R S}{K F}$, mà

$\quad\quad\quad\quad\quad\quad\quad R S=R D-S D=\frac{D E-D F}{2}=\frac{E F}{2}=K F$

Suy ra $R H_1=K I$, mà $R H_1 | K I$ (do cùng vuông góc với $E F$ ) nên $I K R H_1$ là hình chữ nhật, kéo theo $I H_1 | E F$. Một cách tương tự, ta có $I H_2 | E F$ vậy nên đường thẳng $H_1 H_2$ đi qua $I$.

Nhận xét. Trong câu a, tính chất $A, X, Y$ thẳng hàng của bài toán cũng đúng khi thay $M, N$ là chân các đường phân giác bởi cặp điểm liên hợp điều hòa bất kỳ với $B, C$. Điều này có được nhờ tính chất của các đường đối cực (hoặc có thể chứng minh nhờ việc sử dụng phép chiếu trực giao các chùm điều hòa).